So la risposta ma non so come si dimostra.

Messaggioda Erasmus_First » 27/01/2015, 12:51

«Dato un poligono regolare, quanto vale il prodotto delle distanze di un vertice da tutti gli altri?»

Si tratta di un vecchio quiz che suppongo sia già circolato qui in "Matematicamente".

Detti r il raggio del cerchio circoscritto ed n il numero di lati, la risposta è
$n·r^(n–1)$
(come si trova ... "sperimentalmente" provando per r = 1 ed n = 3, 4, 5, 6; e con opportuno programmino ad hoc anche per altri n ...a piacere).

Confesso, però, di non essere [per ora!] riuscito a dimostrare "teoricamente" il risultato in generale (per n indeterminato).

Posto (per comodità) unitario il raggio del cerchio circoscritto e detto n il numero di lati, risulta subito che il cercato prodotto è:
P(n) = [2^(n-1)]·<prodotto, per k da 1 a n -1, di sin(kπ/n)>. (*)

Evidentemente il succo del quiz sta nel calcolare, in generale, (per ogni n intero maggiore di 2), il prodotto
n-1
sin(kπ/n) .
k=1

Come ho detto, se si prova per n = 3, 4, 5 e 6, il prodotto (*) dà sorprendentemente:
P( 3) = 3; P(4) = 4; P(5) = 5; P(6)=6.
Allora viene il sospetto che in generale sia proprio P(n) = n.
E questo risultato mi è confermato "sperimentalmente" per qualsiasi n dal programma "Grapher" (per APPLE in OS X).

Qualcuno sa dimostrare "teoricamente" che è proprio così?
––––––
Immagine
Immagine
Ultima modifica di Erasmus_First il 28/01/2015, 13:27, modificato 2 volte in totale.
Avatar utente
Erasmus_First
Senior Member
Senior Member
 
Messaggio: 79 di 1805
Iscritto il: 11/12/2014, 11:41

Re: So la risposta ma non so come si dimostra.

Messaggioda orsoulx » 27/01/2015, 16:04

Scrivo un'idea, tutta da verificare, perché sulla convergenza nel campo complesso, il più è ormai dimenticato e mi manca la voglia di rinfrescare.
Testo nascosto, fai click qui per vederlo
se $ lim_(z -> 1) |(z^n-1)/(z-1)|=n $, il resto è in discesa.
L'equazione $ z^n-1=0 $ si può scomporre nella produttoria dei binomi $ z-z_k $ dove $ z_k $ è la k-esima soluzione.
Dividendo ambo i membri per $ z-1 $, prendendo i moduli dei due membri e facendone il limite per z tendente a 1, il primo membro diventa il limite che ho scritto sopra ed il secondo $ 2^(n-1) $ per il prodotto che stai cercando.


Ciao
Stephen Wolfram non mi è simpatico, anche perché il malefico Wolfram|Alpha non mi permette di credere che $ e^\pi=(640320^3+744)^(1/\sqrt(163)) $.
"Sono venticinque secoli che la filosofia inquadra i problemi, ma non scatta mai la foto.” - Edoardo Boncinelli, L'infinito in breve.
orsoulx
Cannot live without
Cannot live without
 
Messaggio: 24 di 3906
Iscritto il: 30/12/2014, 11:13

Re: So la risposta ma non so come si dimostra.

Messaggioda Erasmus_First » 28/01/2015, 04:01

@ orsoulx
Vedo che ... "mi curi" (come dicono a Milano). E questo mi fa piacere.
Testo nascosto, fai click qui per vederlo
Aveo replicato (anch'io con "spoiler") in dialogo con te ma non ancora inviato. Non so poi per quale errata manovra si è chiusa la finestra ... e non ho più trovato quel che avevo scritto-
Ma è meglio così: era una lungaggine in cui mi sforzavo di ricordare quel che avevo fatto l'anno scorso (che aveva a che fare con quel che mi dici ora ... ma dall'anno scorso ad oggi la mia memoria è diventata un colabrodo!)

Quel che ricordo è che nel singolo esempio (ero arrivato a n = 13) riuscivo a trasformare quel prodotto di funzioni circolari in somma di funzioni circolari [si dice prostaferesi? Non sono più sicuro di niente!], ma non ero riuscito a farlo in generale

Speravo che qualcuno avesse in tasca una dimostrazione semplice e diretta (per esempio "per induzione" su n).
Ciao ciao.
––––––––––––
Immagine
Immagine
Avatar utente
Erasmus_First
Senior Member
Senior Member
 
Messaggio: 80 di 1805
Iscritto il: 11/12/2014, 11:41

Re: So la risposta ma non so come si dimostra.

Messaggioda milizia96 » 28/01/2015, 13:54

Se si indica con $z$ una radice $n$-esima (primitiva) dell'unità, allora
$$x^{n-1} + x^{n-2} + \dots + x + 1 = (x-z)(x-z^2)\dots (x-z^{n-1})$$
perché, se si moltiplicano entrambi i membri per $(x-1)$ si ottiene lo stesso polinomio (vale a dire $x^n-1$) sia a destra che a sinistra.

Se il raggio del poligono è lungo $1$, allora a noi interessa calcolare
$$\prod_{i=1}^{n-1}\left|1-z^i\right| = \prod_{i=1}^{n-1}\sqrt{(1-z^i)(1-z^{n-i})} = \prod_{i=1}^{n-1}(1-z^i)$$
La prima uguaglianza vale perché $z^i$ e $z^{n-i}$ sono uno il coniugato dell'altro.
L'ultima uguaglianza si ottiene "riordinando" i termini.

L'ultima produttoria è uguale a $n$ (basta porre $x=1$ nell'identità scritta all'inizio).
Avatar utente
milizia96
Senior Member
Senior Member
 
Messaggio: 526 di 1132
Iscritto il: 28/11/2010, 20:39
Località: Mesagne(BR)


Torna a Scervelliamoci un po'

Chi c’è in linea

Visitano il forum: Nessuno e 1 ospite